Exclusion-inclusion formula in probability

Click For Summary
The discussion focuses on proving the inclusion-exclusion formula for probability spaces, specifically for events A1, A2,..., An. The initial proof for n=2 is established by expressing the union of two events in terms of their intersections and complements, leading to the correct probability expression. The challenge arises when attempting to extend the proof to n+1 events, where the user defines new events and attempts to relate the probabilities of these unions and intersections. Suggestions are made to simplify the approach by examining smaller cases before tackling the general case, emphasizing the importance of understanding the pattern in the induction process. The discussion highlights the complexity of the proof and the need for a clearer reduction method to reach the desired formula.
mahler1
Messages
217
Reaction score
0

Homework Statement



Let ##(\Omega,\mathcal F, P)## be a probability space and let ##A_1,A_2,...,A_n## be events in ##\mathcal F##.

Prove the following inclusion-exclusion formula

##P(\bigcup_{i=1}^nA_i)=\sum_{k=1}^n## ##\sum_{\mathcal J \subset \{1,...,n\}; |\mathcal J|=k} (-1)^{k+1}P(\bigcap_{i \in \mathcal J} A_i)##


The Attempt at a Solution



I am trying to prove this formula by induction; for ##n=2##, let ##A, B## be two events in ##\mathcal F##. We can write ##A=(A \setminus B) \cup (A\cap B)##, ##B=(B \setminus A) \cup (A\cap B)##, since these are disjoint unions, then

##P(A)=P(A \setminus B)+P(A\cap B)## and ##P(B)=P(B \setminus A)+P(A\cap B)##.

Now, ##A \cup B## can be expressed as ##A \cup B= (A\setminus B) \cup (A \cap B) \cup (B \setminus A)##, which it's also union of disjoint sets, so

##P(A \cup B)=P(A \setminus B)+P(A\cap B)+P(B \setminus A)=P(A)+P(B)-P(A \cap B)##.

This proves that for ##n=2##, the formula is correct.

Now, I want to show that the formula is true for ##n \implies## the formula is satisfied for ##n+1##

I define ##B_1=A_1,\ldots,B_{n-1}=A_{n-1},B_n=A_n\cup A_{n+1}##. We have

##P(\cup_{i=1}^n B_i)=\sum_{\mathcal J \subset \{1,...,n\}; |\mathcal J|=k} (-1)^{k+1}P(\bigcap_{i \in \mathcal J} B_i)##.

This equals to ##\sum_{\mathcal J \subset \{1,...,n-1\}; |\mathcal J|=k} (-1)^{k+1}P(\bigcap_{i \in \mathcal J} A_i)+\sum_{\mathcal J \subset \{1,...,n-1\}; |\mathcal J|=k-1} (-1)^{k}P(\bigcap_{i \in \mathcal J} A_i \cap (A_n \cup A_{n+1}))##

##=\sum_{\mathcal J \subset \{1,...,n-1\}; |\mathcal J|=k} (-1)^{k+1}P(\bigcap_{i \in \mathcal J} A_i)+\sum_{\mathcal J \subset \{1,...,n-1\}; |\mathcal J|=k-1} (-1)^k(P(\bigcap_{i \in \mathcal J} A_i \cap A_n)+ P(\bigcap_{i \in \mathcal J} A_i \cap A_{n+1})-P(\bigcap_{i \in \mathcal J} A_i \cap A_n \cap A_{n+1}))##.

I don't know how to reduce this expression to the one I need to, which is:

##\sum_{\mathcal J \subset \{1,...,n+1\}; |\mathcal J|=k} (-1)^{k+1}P(\bigcap_{i \in \mathcal J} A_i)##


I would really appreciate some help to finish the problem.
 
Physics news on Phys.org
mahler1 said:

Homework Statement



Let ##(\Omega,\mathcal F, P)## be a probability space and let ##A_1,A_2,...,A_n## be events in ##\mathcal F##.

Prove the following inclusion-exclusion formula

##P(\bigcup_{i=1}^nA_i)=\sum_{k=1}^n## ##\sum_{\mathcal J \subset \{1,...,n\}; |\mathcal J|=k} (-1)^{k+1}P(\bigcap_{i \in \mathcal J} A_i)##


The Attempt at a Solution



I am trying to prove this formula by induction; for ##n=2##, let ##A, B## be two events in ##\mathcal F##. We can write ##A=(A \setminus B) \cup (A\cap B)##, ##B=(B \setminus A) \cup (A\cap B)##, since these are disjoint unions, then

##P(A)=P(A \setminus B)+P(A\cap B)## and ##P(B)=P(B \setminus A)+P(A\cap B)##.

Now, ##A \cup B## can be expressed as ##A \cup B= (A\setminus B) \cup (A \cap B) \cup (B \setminus A)##, which it's also union of disjoint sets, so

##P(A \cup B)=P(A \setminus B)+P(A\cap B)+P(B \setminus A)=P(A)+P(B)-P(A \cap B)##.

This proves that for ##n=2##, the formula is correct.

Now, I want to show that the formula is true for ##n \implies## the formula is satisfied for ##n+1##

I define ##B_1=A_1,\ldots,B_{n-1}=A_{n-1},B_n=A_n\cup A_{n+1}##. We have

##P(\cup_{i=1}^n B_i)=\sum_{\mathcal J \subset \{1,...,n\}; |\mathcal J|=k} (-1)^{k+1}P(\bigcap_{i \in \mathcal J} B_i)##.

This equals to ##\sum_{\mathcal J \subset \{1,...,n-1\}; |\mathcal J|=k} (-1)^{k+1}P(\bigcap_{i \in \mathcal J} A_i)+\sum_{\mathcal J \subset \{1,...,n-1\}; |\mathcal J|=k-1} (-1)^{k}P(\bigcap_{i \in \mathcal J} A_i \cap (A_n \cup A_{n+1}))##

##=\sum_{\mathcal J \subset \{1,...,n-1\}; |\mathcal J|=k} (-1)^{k+1}P(\bigcap_{i \in \mathcal J} A_i)+\sum_{\mathcal J \subset \{1,...,n-1\}; |\mathcal J|=k-1} (-1)^k(P(\bigcap_{i \in \mathcal J} A_i \cap A_n)+ P(\bigcap_{i \in \mathcal J} A_i \cap A_{n+1})-P(\bigcap_{i \in \mathcal J} A_i \cap A_n \cap A_{n+1}))##.

I don't know how to reduce this expression to the one I need to, which is:

##\sum_{\mathcal J \subset \{1,...,n+1\}; |\mathcal J|=k} (-1)^{k+1}P(\bigcap_{i \in \mathcal J} A_i)##


I would really appreciate some help to finish the problem.

Instead of trying to jump immediately to the general case, first try the simple cases of going from n=2 to n=3 and from n=3 to n=4 (and no, I am not joking). That way, you will see what is happening, and be able to grasp how to go the the general case of n to (n+1).
 
  • Like
Likes 1 person
Question: A clock's minute hand has length 4 and its hour hand has length 3. What is the distance between the tips at the moment when it is increasing most rapidly?(Putnam Exam Question) Answer: Making assumption that both the hands moves at constant angular velocities, the answer is ## \sqrt{7} .## But don't you think this assumption is somewhat doubtful and wrong?

Similar threads

  • · Replies 12 ·
Replies
12
Views
2K
  • · Replies 2 ·
Replies
2
Views
1K
  • · Replies 5 ·
Replies
5
Views
3K
  • · Replies 8 ·
Replies
8
Views
2K
  • · Replies 1 ·
Replies
1
Views
1K
  • · Replies 5 ·
Replies
5
Views
2K
  • · Replies 5 ·
Replies
5
Views
2K
  • · Replies 5 ·
Replies
5
Views
2K
  • · Replies 6 ·
Replies
6
Views
2K
Replies
2
Views
2K